Can 3/42 be simplified?

Answers

Answer 1
Answer: Yes, to 1/14
Divide the top and bottom number by 3
Answer 2
Answer: Yes 3/42 can be simplified to 1/14
3/3         1
42/3       14
I hope this helps;)

Related Questions

How do you break apart 3 x 5
At a garden center grass seed sells for eight dollars per pound Kelly Lee spend $10 on grass seed what amount of seed did he buy
The coordinates of the vertices of a triangle are (3,6),(5,0),and (0,0) what is the area of the triangle?
Find the area of the figure. 13 m 6 m 8 mm 4 m
I need help I’m not sure

Jenn will use 18 connecting cubes to make a model of a park. The model will be in the shape of a rectangle and will have a height of one cube. In how many different ways can Jenn make the model of the park?

Answers

This question is basically: how many different pairs of numbers multiply up to 18?

(This is a simpler form of the question because the question is just asking how many shapes of a rectangle can Jenn create with 18 blocks)

So, the numbers that multiply up to 18 are:

1 x 18
2 x 9
3 x 6

So, the answer is: Jenn can make the model of the park in 3 different ways.

Write the formula for a triangle with a height of 2 1/2 and a 13 in base

Answers


There's actually no such thing as "the formula for a triangle". 
There are many details of a triangle that you might be asked
to calculate.  You might be asked to find the perimeter, the area,
the angles, or the length of one or more sides.  Your question
doesn't specify which of these things we're expected to find.


Simplify the ratio 24/56

Answers

How to solve:
Step 1 :  Note down the given ratio from the questions.

Step 2 :  Find the greatest common factor, and divide the numerator and denominator by that value.
Answer: 3:7

A cookie is chosen from a jar containing 4 choc chip, 3 pb, 2 oatmeal.  What is the probability that you will choose a choc chip.  Express your answer as a fraction in lowest form.

Answers

|\Omega|=9\n |A|=4\n P(A)=(|A|)/(|\Omega|)=(4)/(9)

Round 131.625 to the nearest tenth

Answers

131.625 to the nearest tenth is 132
The answer is 131.6 :-)

(k^4_3_3k^3)+(-5k^3+6k^3_8k^5)

Answers

Answer:

(k4+3+3k3)+(-5k3+6k3+8k5)

Final result :

 8k5 + k4 + 4k3 + 3

Step by step solution :

Step  1  :

Equation at the end of step  1  :

 (((k4)+3)+(3•(k3)))+(((0-(5•(k3)))+(6•(k3)))+23k5)

Step  2  :

Equation at the end of step  2  :

 (((k4)+3)+(3•(k3)))+(((0-(5•(k3)))+(2•3k3))+23k5)

Step  3  :

Equation at the end of step  3  :

 (((k4)+3)+(3•(k3)))+(((0-5k3)+(2•3k3))+23k5)

Step  4  :

Equation at the end of step  4  :

 (((k4) +  3) +  3k3) +  (8k5 + k3)

Step  5  :

Checking for a perfect cube :

5.1    8k5+k4+4k3+3  is not a perfect cube


Trying to factor by pulling out :

5.2      Factoring:  8k5+k4+4k3+3


Thoughtfully split the expression at hand into groups, each group having two terms :


Group 1:  k4+3

Group 2:  8k5+4k3


Pull out from each group separately :


Group 1:   (k4+3) • (1)

Group 2:   (2k2+1) • (4k3)


Bad news !! Factoring by pulling out fails :


The groups have no common factor and can not be added up to form a multiplication.


Polynomial Roots Calculator :

5.3    Find roots (zeroes) of :       F(k) = 8k5+k4+4k3+3

Polynomial Roots Calculator is a set of methods aimed at finding values of  k  for which   F(k)=0  


Rational Roots Test is one of the above mentioned tools. It would only find Rational Roots that is numbers  k  which can be expressed as the quotient of two integers


The Rational Root Theorem states that if a polynomial zeroes for a rational number  P/Q   then  P  is a factor of the Trailing Constant and  Q  is a factor of the Leading Coefficient


In this case, the Leading Coefficient is  8  and the Trailing Constant is  3.


The factor(s) are:


of the Leading Coefficient :  1,2 ,4 ,8

of the Trailing Constant :  1 ,3


Let us test ....


  P    Q    P/Q    F(P/Q)     Divisor

     -1       1        -1.00        -8.00    

     -1       2        -0.50        2.31    

     -1       4        -0.25        2.93    

     -1       8        -0.13        2.99    

     -3       1        -3.00       -1968.00    

     -3       2        -1.50        -66.19    

     -3       4        -0.75        -0.27    

     -3       8        -0.38        2.75    

     1       1        1.00        16.00    

     1       2        0.50        3.81    

     1       4        0.25        3.07    

     1       8        0.13        3.01    

     3       1        3.00        2136.00    

     3       2        1.50        82.31    

     3       4        0.75        6.90    

     3       8        0.38        3.29    


Polynomial Roots Calculator found no rational roots


Final result :

 8k5 + k4 + 4k3 + 3

Step-by-step explanation: